Violation of Newton's Third Law

 

Violation of Newton's Third Law


আমাদের ক্লাস ৯ থেকেই নিউটনের গতিসূত্রের সাথে পরিচয় হয়।নিউটনের সূত্রগুলোর মধ্যে তৃতীয় সূত্রটা সবচেয়ে বেশি জনপ্রিয়।সেটা মোটামুটি আমরা সবাই কমবেশি জানি।আচ্ছা এমন কি হতে পারে-নিউটনের এই থার্ড ল কাজ করছে না?একটা ক্রিয়ার বিপরীত দিকে তার প্রতিক্রিয়া নেই?নিউটনের সূত্র কি ভায়োলেট হতে পারে?


….

হ্যাঁ,পারে।সেটা নিয়েই আজকের আর্টিকেল।ইলেক্ট্রোডায়নামিক্সে নিউটনের থার্ড ল ভায়োলেট হতে পারে। এই আর্টিকেলটা  একটু দুর্বোধ্য হতে পারে।তবে,একটু চিন্তা করলেই ক্লিয়ার হয়ে যাবে।এইটা পড়ার আগে দুইটা জিনিস সম্পর্কে জানলে ভালো হয়।দুইটা ভেক্টরের ক্রস প্রোডাক্ট আর বায়ো স্যাভার্ট ল।তবে,বায়ো স্যাভার্ট ম্যান্ডেটরি না।তবে,ক্রস প্রোডাক্ট বুঝতেই হবে।আর একটা ইম্পর্ট্যান্ট বিষয়-তল ব্যাপারটা বোঝা।   



এখন দেখা যাক ব্যাপারটা।ধরি,xy তলে x অক্ষ বরাবর নেগেটিভ দিকে +q1 চার্জ v বেগে যাচ্ছে।আবার,আরেকটা +q2 চার্জ y অক্ষ বরাবর নেগেটিভ দিকে একই v বেগে যাচ্ছে।

এখন,

q1 থেকে q2  এর সরাসরি  দূরত্ব ভেক্টর,r(12)=-i+j 

[ভেক্টরের ত্রিভুজ সূত্র এপ্লাই করেছি।আর কিছু না।আর আমরা শুধু দিক নিয়ে কাজ করছি এখানে।]


একইভাবে,

q2 থেকে q1 এর সরাসরি দূরত্ব ভেক্টর লিখতে পারি,r(21)=-j+i


এখন,

গতিশীল q1 চার্জের কারণে একটা ম্যাগনেটিক ফিল্ড সৃষ্টি হবে।সেটার দিক হবে,বেগ এবং r(12) এর ক্রস প্রোডাক্টের দিকের উপর।কারণটা হলো-বায়ো স্যাভার্ট সূত্র থেকে জানা যায়। 


q1 চার্জের দরুণ সৃষ্ট ম্যাগনেটিক ফিল্ড,

B1 এর দিক=(-i)×r(12) [বেগের দিক × দূরত্ব ভেক্টর]

 =(-i)×(-i+j)

=(-i×-i)+(-i×j)

=o+j×i

=-k

তারমানে,q1 চার্জের জন্য ম্যাগনেটিক ফিল্ড এর দিক -k বরাবর।

যাদের এই অন্ধ ম্যাথটা বুঝতে কষ্ট হচ্ছে,তারা নবম-দশম শ্রেণির বইয়ের ডানহাতি নিয়ম ব্যবহার করতে পারবেন।দিকটা যদি আরও ক্লিয়ারলি বুঝাতে চাই তাহলে,যেহেতু ক্লকওয়াইজ ঘূর্ণন,তাই xy তলের নিচের দিকে।


তাহলে,

গতিশীল q2 চার্জের জন্য সৃষ্ট ম্যাগনেটিক ফিল্ড,

B2 এর দিক =(-j)×(-j+i)

=(-j×-j)+(-j×i)

=0+(i×j)

=k

আগেরটার ব্যাখা বুঝে থাকলে এটারটাও বুঝবেন আশা করি।ডান হাতের,বাম হাতের নিয়ম দিয়েও বের করতে পারবেন।দিকটা xy তলের উপরের দিকে।


এখন,ব্যাপারটা একটু মনোযোগ দিয়ে পড়ুন-

q1 এর জন্য সৃষ্ট ম্যাগনেটিক ফিল্ড B1 এর জন্য q2 চার্জ একটা ম্যাগনেটিক ফোর্স F(21) অনুভব করবে।কেউ ব্যাপারটা না জানলে লরেন্টজ  ফর্মুলা নিয়ে পড়ে পারেন।F=q(v×B) এই ফর্মুলা থেকে পাওয়া যায় সেটার দিক হবে বেগের দিক এবং ম্যাগনেটিক ফিল্ড এর দিকের ক্রস প্রোডাক্টের দিকের উপর।


তাহলে,F(21) এর দিক,

=(-j)×(-k)

=i

তারমানে,q1 চার্জের ম্যাগনেটিক ফিল্ড এর জন্য q2 চার্জ বল অনুভব করবে i মানে x অক্ষ বরাবর।এটা না বুঝলেও স্ক্রু নিয়ম,ডানহাতি,বামহাতি তিন আঙ্গুলের নিয়ম

যেভাবে খুশি সেভাবে বের করতে পারবেন।



একইভাবে,

q2 চার্জের কারণে সৃষ্ট ম্যাগনেটিক ফিল্ড এর কারণে q1 চার্জ ম্যাগনেটিক ফোর্স F(12) ফিল করবে।    

F(12) এর দিক,

=(-i)×(k)

=k×i

=j


এইটাই হাজারটা নিয়মে বের করতে পারবেন।দুই একটা নিয়মের ছবিও দিয়ে দিয়েছি।তারমানে,q2 চার্জের ম্যাগনেটিক ফিল্ড এর জন্য q1 চার্জ বল অনুভব করবে j মানে y অক্ষ বরাবর।    


এখন,ধরি,

দুইটা চার্জের মান সমান।q1=q2

তাদের বেগের মানও সমান।

সেই অনুয়ায়ী B1 এবং B2 এর মানও সমান।

তাহলে,|F(12)|=|F(21)|

তারমানে,দুইটা চার্জের একে অপরের উপর প্রযুক্ত চৌম্বক বলের মান সমান।ক্লিয়ার এতটুকু?


এখন,আমরা সবাই জানি,নিউটনের থার্ড ল এর বিবৃতিটা এরকম-

"প্রত্যেক ক্রিয়ারই একটা সমান ও বিপরীত প্রতিক্রিয়া আছে।"

মানে,A জিনিস যদি B জিনিসকে F বল দেয়।তাহলে,B জিনিসও A জিনিসকে F বল দিবে।আর দুইটা বলের দিক হবে সম্পূর্ণ বিপরীত।


এখন,উপরের চার্জের কাহিনীতে কি দেখা যায়,দুইটা চার্জ একে অপরকে ম্যাগনেটিক ফোর্স দিচ্ছে।দুইটা ফোর্সেরই মান সমান হচ্ছে।কিন্তু দিক কি আসলে বিপরীত?q2 চার্জ q1 এর জন্য বল পাচ্ছে x অক্ষ বরাবর।আর q1 চার্জ q2 এর জন্য বল পাচ্ছে y অক্ষ বরাবর।তার মানে কি?এখানে দুইটা বল মোটেও বিপরীত না।একটা আরেকটার উপর লম্ব।এখানেই ঘটে গেলো-এক আধিভৌতিক কাহিনী-নিউটনের থার্ড ল এর ভায়োলেশন।


[নোট:জটিলতা এড়াতে এখানে ইলেক্ট্রিক ফিল্ডের ব্যাপারটা ইগ্নোর করেছিলাম।সেটা,মূল বিষয়ে তেমন কোনো ইফেক্ট ফেলে না। যেহেতু কমেন্ট বক্সে সেটা নিয়ে প্রশ্ন এসেছে-তাই বলছি-রেফারেন্সে একটা ভিডিওর লিঙ্ক দিয়েছিলাম।সেখানে,ইলেক্ট্রিক ফিল্ডের ব্যাপারটা ক্লিয়ার করেছে।সাথে ভরবেগের ব্যাপার নিয়েও পপ কিছু বলেছে।তাই,পোস্ট পড়ার পর ভিডিওটা দেখার অনুরোধ করছি।সেখান থেকে ইন্টুইটিভ একটা ছবিও এড করে দিলাম।]   



তারমানে কি?সমকোণে গতিশীল দুইটা চার্জের পারস্পরিক চৌম্বক বল নিউটনের তৃতীয় সূত্রকে মানে না।



হিসাব মিললো কি?


আসলেই কি এই জায়গায় থার্ড ল খাটে না?আমি তো কোনো ভুল করিনি।কি হতো যদি q1 আর q2 সমান না হতো?থার্ড ল এর জেনারালাইজড রূপ কি কিছু নেই?সেটা কি এখানে খাটবে না?রকেটে যেমন খাটে।


ধন্যবাদ।   


Reference:


1)

 https://www.google.com/amp/s/bigganblog.org/2020/04/%25E0%25A6%2595%25E0%25A7%258D%25E0%25A6%25B2%25E0%25A6%25BE%25E0%25A6%25B8%25E0%25A6%25BF%25E0%25A6%2595%25E0%25A7%258D%25E0%25A6%25AF%25E0%25A6%25BE%25E0%25A6%25B2-%25E0%25A6%25AE%25E0%25A7%2587%25E0%25A6%2595%25E0%25A6%25BE%25E0%25A6%25A8%25E0%25A6%25BF%25E0%25A6%2595%25E0%25A7%258D%25E0%25A6%25B8-%25E0%25A7%25A7/amp/


2)https://physics.stackexchange.com/questions/114466/apparent-violation-of-newtons-3-textrd-law-and-the-conservation-of-momen


3)https://youtu.be/_ggblMjsbVE


4)https://en.m.wikipedia.org/wiki/Weber_electrodynamics


5)বায়ো স্যাভার্ট ল:

https://en.m.wikipedia.org/wiki/Biot%E2%80%93Savart_law


4)Cross Product কেউ না বুঝলে- https://www.mathsisfun.com/algebra/vectors-cross-product.html





  


       

  

        


  



Popular Posts